Inequality in 3 variables with a constraint condition












2












$begingroup$


To prove : a(b+c)/bc + b(c+a)/ca + c(a+b)/ab > 2/(ab+bc+ca) where a+b+c=1 and a,b,c are positive real numbers



Here's my way : Add 1 to each summand in the LHS and subtract 3 (=1+1+1) and after some algebraic manipulation I got (ab+bc+ac)/abc > 2/(ab+bc+ac) +3 which again after some algebra reduces to a²b²+b²c²+a²c² +2abc(a+b+c) > 2+3abc(a+b+c)=> a²b²+b²c²+a²c² > 2+abc . Now, this obviously true for positive a,b,c but I can't prove it rigorously this.










share|cite|improve this question











$endgroup$

















    2












    $begingroup$


    To prove : a(b+c)/bc + b(c+a)/ca + c(a+b)/ab > 2/(ab+bc+ca) where a+b+c=1 and a,b,c are positive real numbers



    Here's my way : Add 1 to each summand in the LHS and subtract 3 (=1+1+1) and after some algebraic manipulation I got (ab+bc+ac)/abc > 2/(ab+bc+ac) +3 which again after some algebra reduces to a²b²+b²c²+a²c² +2abc(a+b+c) > 2+3abc(a+b+c)=> a²b²+b²c²+a²c² > 2+abc . Now, this obviously true for positive a,b,c but I can't prove it rigorously this.










    share|cite|improve this question











    $endgroup$















      2












      2








      2





      $begingroup$


      To prove : a(b+c)/bc + b(c+a)/ca + c(a+b)/ab > 2/(ab+bc+ca) where a+b+c=1 and a,b,c are positive real numbers



      Here's my way : Add 1 to each summand in the LHS and subtract 3 (=1+1+1) and after some algebraic manipulation I got (ab+bc+ac)/abc > 2/(ab+bc+ac) +3 which again after some algebra reduces to a²b²+b²c²+a²c² +2abc(a+b+c) > 2+3abc(a+b+c)=> a²b²+b²c²+a²c² > 2+abc . Now, this obviously true for positive a,b,c but I can't prove it rigorously this.










      share|cite|improve this question











      $endgroup$




      To prove : a(b+c)/bc + b(c+a)/ca + c(a+b)/ab > 2/(ab+bc+ca) where a+b+c=1 and a,b,c are positive real numbers



      Here's my way : Add 1 to each summand in the LHS and subtract 3 (=1+1+1) and after some algebraic manipulation I got (ab+bc+ac)/abc > 2/(ab+bc+ac) +3 which again after some algebra reduces to a²b²+b²c²+a²c² +2abc(a+b+c) > 2+3abc(a+b+c)=> a²b²+b²c²+a²c² > 2+abc . Now, this obviously true for positive a,b,c but I can't prove it rigorously this.







      inequality contest-math cauchy-schwarz-inequality






      share|cite|improve this question















      share|cite|improve this question













      share|cite|improve this question




      share|cite|improve this question








      edited Jan 9 at 22:13









      Michael Rozenberg

      101k1591193




      101k1591193










      asked Jan 9 at 16:31









      Epsilon zeroEpsilon zero

      35118




      35118






















          2 Answers
          2






          active

          oldest

          votes


















          2












          $begingroup$

          If we arrange
          $$
          frac{ab+bc+ac}{abc} ge frac{2}{ab+bc+ac} +3
          $$
          then we have
          $$
          (ab+bc+ac)^2 =sum_{text{cyc}}a^2b^2 +2abcge abc(2+3(ab+bc+ac))=2abc +3sum_{text{cyc}}a^2b^2c.
          $$
          We need to show
          $$
          sum_{text{cyc}}a^2b^2ge 3sum_{text{cyc}}a^2b^2c.
          $$
          This is equivalent to
          $$
          sum_{text{cyc}}a^2b^2(a+b+c)ge 3sum_{text{cyc}}a^2b^2c.
          $$
          By rearrangement, we have
          $$
          sum_{text{cyc}}a^3b^2ge sum_{text{cyc}}a^2b^2c,
          $$
          and
          $$
          sum_{text{cyc}}a^2b^3ge sum_{text{cyc}}a^2b^2c.
          $$
          This proves the inequality.



          Another approach: We need to show
          $$
          sum_{text{cyc}}frac{a}{b}+sum_{text{cyc}}frac{a}{c}ge frac{2}{ab+bc+ca}.
          $$
          By C-S, we have
          $$
          sum_{text{cyc}}frac{a}{b}sum_{text{cyc}}abge left(sum_{text{cyc}}aright)^2 =1,
          $$
          and
          $$
          sum_{text{cyc}}frac{a}{c}sum_{text{cyc}}acge left(sum_{text{cyc}}aright)^2 =1.
          $$






          share|cite|improve this answer











          $endgroup$





















            0












            $begingroup$

            Because by C-S
            $$(ab+ac+bc)sum_{cyc}frac{a(b+c)}{bc}=sum_{cyc}acsum_{cyc}frac{a}{c}+sum_{cyc}absum_{cyc}frac{a}{b}geq$$
            $$geq(a+b+c)^2+(a+b+c)^2=2.$$






            share|cite|improve this answer









            $endgroup$













              Your Answer





              StackExchange.ifUsing("editor", function () {
              return StackExchange.using("mathjaxEditing", function () {
              StackExchange.MarkdownEditor.creationCallbacks.add(function (editor, postfix) {
              StackExchange.mathjaxEditing.prepareWmdForMathJax(editor, postfix, [["$", "$"], ["\\(","\\)"]]);
              });
              });
              }, "mathjax-editing");

              StackExchange.ready(function() {
              var channelOptions = {
              tags: "".split(" "),
              id: "69"
              };
              initTagRenderer("".split(" "), "".split(" "), channelOptions);

              StackExchange.using("externalEditor", function() {
              // Have to fire editor after snippets, if snippets enabled
              if (StackExchange.settings.snippets.snippetsEnabled) {
              StackExchange.using("snippets", function() {
              createEditor();
              });
              }
              else {
              createEditor();
              }
              });

              function createEditor() {
              StackExchange.prepareEditor({
              heartbeatType: 'answer',
              autoActivateHeartbeat: false,
              convertImagesToLinks: true,
              noModals: true,
              showLowRepImageUploadWarning: true,
              reputationToPostImages: 10,
              bindNavPrevention: true,
              postfix: "",
              imageUploader: {
              brandingHtml: "Powered by u003ca class="icon-imgur-white" href="https://imgur.com/"u003eu003c/au003e",
              contentPolicyHtml: "User contributions licensed under u003ca href="https://creativecommons.org/licenses/by-sa/3.0/"u003ecc by-sa 3.0 with attribution requiredu003c/au003e u003ca href="https://stackoverflow.com/legal/content-policy"u003e(content policy)u003c/au003e",
              allowUrls: true
              },
              noCode: true, onDemand: true,
              discardSelector: ".discard-answer"
              ,immediatelyShowMarkdownHelp:true
              });


              }
              });














              draft saved

              draft discarded


















              StackExchange.ready(
              function () {
              StackExchange.openid.initPostLogin('.new-post-login', 'https%3a%2f%2fmath.stackexchange.com%2fquestions%2f3067665%2finequality-in-3-variables-with-a-constraint-condition%23new-answer', 'question_page');
              }
              );

              Post as a guest















              Required, but never shown

























              2 Answers
              2






              active

              oldest

              votes








              2 Answers
              2






              active

              oldest

              votes









              active

              oldest

              votes






              active

              oldest

              votes









              2












              $begingroup$

              If we arrange
              $$
              frac{ab+bc+ac}{abc} ge frac{2}{ab+bc+ac} +3
              $$
              then we have
              $$
              (ab+bc+ac)^2 =sum_{text{cyc}}a^2b^2 +2abcge abc(2+3(ab+bc+ac))=2abc +3sum_{text{cyc}}a^2b^2c.
              $$
              We need to show
              $$
              sum_{text{cyc}}a^2b^2ge 3sum_{text{cyc}}a^2b^2c.
              $$
              This is equivalent to
              $$
              sum_{text{cyc}}a^2b^2(a+b+c)ge 3sum_{text{cyc}}a^2b^2c.
              $$
              By rearrangement, we have
              $$
              sum_{text{cyc}}a^3b^2ge sum_{text{cyc}}a^2b^2c,
              $$
              and
              $$
              sum_{text{cyc}}a^2b^3ge sum_{text{cyc}}a^2b^2c.
              $$
              This proves the inequality.



              Another approach: We need to show
              $$
              sum_{text{cyc}}frac{a}{b}+sum_{text{cyc}}frac{a}{c}ge frac{2}{ab+bc+ca}.
              $$
              By C-S, we have
              $$
              sum_{text{cyc}}frac{a}{b}sum_{text{cyc}}abge left(sum_{text{cyc}}aright)^2 =1,
              $$
              and
              $$
              sum_{text{cyc}}frac{a}{c}sum_{text{cyc}}acge left(sum_{text{cyc}}aright)^2 =1.
              $$






              share|cite|improve this answer











              $endgroup$


















                2












                $begingroup$

                If we arrange
                $$
                frac{ab+bc+ac}{abc} ge frac{2}{ab+bc+ac} +3
                $$
                then we have
                $$
                (ab+bc+ac)^2 =sum_{text{cyc}}a^2b^2 +2abcge abc(2+3(ab+bc+ac))=2abc +3sum_{text{cyc}}a^2b^2c.
                $$
                We need to show
                $$
                sum_{text{cyc}}a^2b^2ge 3sum_{text{cyc}}a^2b^2c.
                $$
                This is equivalent to
                $$
                sum_{text{cyc}}a^2b^2(a+b+c)ge 3sum_{text{cyc}}a^2b^2c.
                $$
                By rearrangement, we have
                $$
                sum_{text{cyc}}a^3b^2ge sum_{text{cyc}}a^2b^2c,
                $$
                and
                $$
                sum_{text{cyc}}a^2b^3ge sum_{text{cyc}}a^2b^2c.
                $$
                This proves the inequality.



                Another approach: We need to show
                $$
                sum_{text{cyc}}frac{a}{b}+sum_{text{cyc}}frac{a}{c}ge frac{2}{ab+bc+ca}.
                $$
                By C-S, we have
                $$
                sum_{text{cyc}}frac{a}{b}sum_{text{cyc}}abge left(sum_{text{cyc}}aright)^2 =1,
                $$
                and
                $$
                sum_{text{cyc}}frac{a}{c}sum_{text{cyc}}acge left(sum_{text{cyc}}aright)^2 =1.
                $$






                share|cite|improve this answer











                $endgroup$
















                  2












                  2








                  2





                  $begingroup$

                  If we arrange
                  $$
                  frac{ab+bc+ac}{abc} ge frac{2}{ab+bc+ac} +3
                  $$
                  then we have
                  $$
                  (ab+bc+ac)^2 =sum_{text{cyc}}a^2b^2 +2abcge abc(2+3(ab+bc+ac))=2abc +3sum_{text{cyc}}a^2b^2c.
                  $$
                  We need to show
                  $$
                  sum_{text{cyc}}a^2b^2ge 3sum_{text{cyc}}a^2b^2c.
                  $$
                  This is equivalent to
                  $$
                  sum_{text{cyc}}a^2b^2(a+b+c)ge 3sum_{text{cyc}}a^2b^2c.
                  $$
                  By rearrangement, we have
                  $$
                  sum_{text{cyc}}a^3b^2ge sum_{text{cyc}}a^2b^2c,
                  $$
                  and
                  $$
                  sum_{text{cyc}}a^2b^3ge sum_{text{cyc}}a^2b^2c.
                  $$
                  This proves the inequality.



                  Another approach: We need to show
                  $$
                  sum_{text{cyc}}frac{a}{b}+sum_{text{cyc}}frac{a}{c}ge frac{2}{ab+bc+ca}.
                  $$
                  By C-S, we have
                  $$
                  sum_{text{cyc}}frac{a}{b}sum_{text{cyc}}abge left(sum_{text{cyc}}aright)^2 =1,
                  $$
                  and
                  $$
                  sum_{text{cyc}}frac{a}{c}sum_{text{cyc}}acge left(sum_{text{cyc}}aright)^2 =1.
                  $$






                  share|cite|improve this answer











                  $endgroup$



                  If we arrange
                  $$
                  frac{ab+bc+ac}{abc} ge frac{2}{ab+bc+ac} +3
                  $$
                  then we have
                  $$
                  (ab+bc+ac)^2 =sum_{text{cyc}}a^2b^2 +2abcge abc(2+3(ab+bc+ac))=2abc +3sum_{text{cyc}}a^2b^2c.
                  $$
                  We need to show
                  $$
                  sum_{text{cyc}}a^2b^2ge 3sum_{text{cyc}}a^2b^2c.
                  $$
                  This is equivalent to
                  $$
                  sum_{text{cyc}}a^2b^2(a+b+c)ge 3sum_{text{cyc}}a^2b^2c.
                  $$
                  By rearrangement, we have
                  $$
                  sum_{text{cyc}}a^3b^2ge sum_{text{cyc}}a^2b^2c,
                  $$
                  and
                  $$
                  sum_{text{cyc}}a^2b^3ge sum_{text{cyc}}a^2b^2c.
                  $$
                  This proves the inequality.



                  Another approach: We need to show
                  $$
                  sum_{text{cyc}}frac{a}{b}+sum_{text{cyc}}frac{a}{c}ge frac{2}{ab+bc+ca}.
                  $$
                  By C-S, we have
                  $$
                  sum_{text{cyc}}frac{a}{b}sum_{text{cyc}}abge left(sum_{text{cyc}}aright)^2 =1,
                  $$
                  and
                  $$
                  sum_{text{cyc}}frac{a}{c}sum_{text{cyc}}acge left(sum_{text{cyc}}aright)^2 =1.
                  $$







                  share|cite|improve this answer














                  share|cite|improve this answer



                  share|cite|improve this answer








                  edited Jan 9 at 17:21

























                  answered Jan 9 at 17:14









                  SongSong

                  11.4k628




                  11.4k628























                      0












                      $begingroup$

                      Because by C-S
                      $$(ab+ac+bc)sum_{cyc}frac{a(b+c)}{bc}=sum_{cyc}acsum_{cyc}frac{a}{c}+sum_{cyc}absum_{cyc}frac{a}{b}geq$$
                      $$geq(a+b+c)^2+(a+b+c)^2=2.$$






                      share|cite|improve this answer









                      $endgroup$


















                        0












                        $begingroup$

                        Because by C-S
                        $$(ab+ac+bc)sum_{cyc}frac{a(b+c)}{bc}=sum_{cyc}acsum_{cyc}frac{a}{c}+sum_{cyc}absum_{cyc}frac{a}{b}geq$$
                        $$geq(a+b+c)^2+(a+b+c)^2=2.$$






                        share|cite|improve this answer









                        $endgroup$
















                          0












                          0








                          0





                          $begingroup$

                          Because by C-S
                          $$(ab+ac+bc)sum_{cyc}frac{a(b+c)}{bc}=sum_{cyc}acsum_{cyc}frac{a}{c}+sum_{cyc}absum_{cyc}frac{a}{b}geq$$
                          $$geq(a+b+c)^2+(a+b+c)^2=2.$$






                          share|cite|improve this answer









                          $endgroup$



                          Because by C-S
                          $$(ab+ac+bc)sum_{cyc}frac{a(b+c)}{bc}=sum_{cyc}acsum_{cyc}frac{a}{c}+sum_{cyc}absum_{cyc}frac{a}{b}geq$$
                          $$geq(a+b+c)^2+(a+b+c)^2=2.$$







                          share|cite|improve this answer












                          share|cite|improve this answer



                          share|cite|improve this answer










                          answered Jan 9 at 22:13









                          Michael RozenbergMichael Rozenberg

                          101k1591193




                          101k1591193






























                              draft saved

                              draft discarded




















































                              Thanks for contributing an answer to Mathematics Stack Exchange!


                              • Please be sure to answer the question. Provide details and share your research!

                              But avoid



                              • Asking for help, clarification, or responding to other answers.

                              • Making statements based on opinion; back them up with references or personal experience.


                              Use MathJax to format equations. MathJax reference.


                              To learn more, see our tips on writing great answers.




                              draft saved


                              draft discarded














                              StackExchange.ready(
                              function () {
                              StackExchange.openid.initPostLogin('.new-post-login', 'https%3a%2f%2fmath.stackexchange.com%2fquestions%2f3067665%2finequality-in-3-variables-with-a-constraint-condition%23new-answer', 'question_page');
                              }
                              );

                              Post as a guest















                              Required, but never shown





















































                              Required, but never shown














                              Required, but never shown












                              Required, but never shown







                              Required, but never shown

































                              Required, but never shown














                              Required, but never shown












                              Required, but never shown







                              Required, but never shown







                              Popular posts from this blog

                              Can a sorcerer learn a 5th-level spell early by creating spell slots using the Font of Magic feature?

                              Does disintegrating a polymorphed enemy still kill it after the 2018 errata?

                              A Topological Invariant for $pi_3(U(n))$